Care Exam 4

Ace your homework & exams now with Quizwiz!

7. A nurse assesses a client who is prescribed levothyroxine for hypothyroidism. Which assessment finding alerts the nurse that drug therapy is effective? a. Thirst is recognized and fluid intake is appropriate. b. Weight has been the same for 3 weeks. c. Total white blood cell count is 6000 cells/mm3 (6 109/L). d. Heart rate is 76 beats/min and regular.

ANS: D Hypothyroidism decreases body functioning and can result in effects such as bradycardia, confusion, and constipation. If a client's heart rate is bradycardic while on thyroid hormone replacement, this is an indicator that the replacement may not be adequate. Conversely, a heart rate above 100 beats/min may indicate that the client is receiving too much of the thyroid hormone. Thirst, fluid intake, weight, and white blood cell count do not represent a therapeutic response to this medication.

27. A nurse reviews the laboratory results of a client who is receiving intravenous insulin. Which would alert the nurse to intervene immediately? a. Serum chloride level of 98 mEq/L (98 mmol/L) b. Serum calcium level of 8.8 mg/dL (2.2 mmol/L) c. Serum sodium level of 132 mEq (132 mmol/L) d. Serum potassium level of 2.5 mEq/L (2.5 mmol/L)

ANS: D Insulin activates the sodium-potassium ATPase pump, increasing the movement of potassium from the extracellular fluid into the intracellular fluid, resulting in hypokalemia. In hyperglycemia, hypokalemia can also result from excessive urine loss of potassium. The chloride level is normal. The calcium and sodium levels are slightly low, but this would not be related to hyperglycemia and insulin administration.

21. A nurse teaches a client with type 1 diabetes mellitus. Which statement would the nurse include in this client's teaching to decrease the client's insulin needs? a. "Limit your fluid intake to 2 L a day." b. "Animal organ meat is high in insulin." c. "Limit your carbohydrate intake to 80 g a day." d. "Walk at a moderate pace for 1 mile daily."

ANS: D Moderate exercise such as walking helps regulate blood glucose levels on a daily basis and results in lowered insulin requirements for patients with type 1 diabetes mellitus. Restricting fluids and eating organ meats will not reduce insulin needs. People with diabetes need at least 130 g of carbohydrates each day.

2. A nurse assesses a client who is recovering from a subtotal thyroidectomy and observes the development of stridor. What is the priority action for the nurse to take? a. Apply oxygen via nasal cannula at 2 L/min. b. Document the finding and assess the client hourly. c. Place the client in high-Fowler position in the bed. d. Contact the Rapid Response Team and prepare for intubation.

ANS: D Stridor on exhalation is a hallmark of respiratory distress, usually caused by obstruction resulting from edema. The nurse should prepare to assist with emergency intubation or tracheostomy while notifying the Rapid Response Team. Stridor is an emergency situation; therefore, reassuring the client, documenting, and reassessing in an hour do not address the urgency of the situation. Oxygen should be applied, but this action will not keep the airway open.

35. The nurse is caring for a newly admitted older adult who has a blood glucose of 300 mg/dL (16.7 mmol/L), a urine output of 185 mL in the past 8 hours, and a blood urea nitrogen (BUN) of 44 mg/dL (15.7 mmol/L). What diabetic complication does the nurse suspect? a. Diabetic ketoacidosis (DKA) b. Severe hypoglycemia c. Chronic kidney disease (CKD) d. Hyperglycemic-hyperosmolar state (HHS)

ANS: D The client most likely has diabetes mellitus type 2 and has a high blood glucose causing increased blood osmolarity and dehydration, as evidenced by an insufficient urinary output and increased BUN. Older adults are at the greatest risk for dehydration due to age-related physiologic changes.

13. The nurse is caring for a client with adrenal insufficiency. What priority physical assessment would the nurse perform? a. Respiratory assessment b. Skin assessment c. Neurologic assessment d. Cardiac assessment

ANS: D The client who has adrenal insufficiency has hyperkalemia that can cause cardiac dysrhythmias. Therefore, the nurse would monitor the client's cardiovascular status through frequent assessments.

9. A client is being treated for diabetes insipidus (DI) with synthetic vasopressin (desmopressin). What is the priority health teaching that the nurse provides regarding drug therapy? a. The need to check the client's urinary specific gravity. b. The need to take blood pressure at least twice a day. c. The need to monitor blood glucose every day. d. The need to weigh every day and report weight gain.

ANS: D The client with DI who takes lifelong hormone replacement will need to report significant weight gain to monitor for water toxicity. Water toxicity causes headache, vomiting, and acute confusion.

20. A nurse cares for a client experiencing diabetic ketoacidosis who presents with Kussmaul respirations. What action would the nurse take? a. Administration of oxygen via facemask b. Intravenous administration of 10% glucose c. Implementation of seizure precautions d. Administration of intravenous insulin

ANS: D The rapid, deep respiratory efforts of Kussmaul respirations are the body's attempt to reduce the acids produced by using fat rather than glucose for fuel. Only the administration of insulin will reduce this type of respiration by assisting glucose to move into cells and to be used for fuel instead of fat. The patient who is in ketoacidosis may not experience any respiratory impairment and therefore does not need additional oxygen. Giving the patient glucose would be contraindicated. The patient does not require seizure precautions.

17. The nurse is planning the care of a patient with hyperthyroidism. What should the nurse specify in the patients meal plan? A) A clear liquid diet, high in nutrients B) Small, frequent meals, high in protein and calories C) Three large, bland meals a day D) A diet high in fiber and plant-sourced fat

B Feedback: A patient with hyperthyroidism has an increased appetite. The patient should be counseled to consume several small, well-balanced meals. High-calorie, high-protein foods are encouraged. A clear liquid diet would not satisfy the patients caloric or hunger needs. A diet rich in fiber and fat should be avoided because these foods may lead to GI upset or increase peristalsis.

5. The nurse is caring for a patient with Addisons disease who is scheduled for discharge. When teaching the patient about hormone replacement therapy, the nurse should address what topic? A) The possibility of precipitous weight gain B) The need for lifelong steroid replacement C) The need to match the daily steroid dose to immediate symptoms D) The importance of monitoring liver function

B Feedback: Because of the need for lifelong replacement of adrenal cortex hormones to prevent addisonian crises, the patient and family members receive explicit education about the rationale for replacement therapy and proper dosage. Doses are not adjusted on a short-term basis. Weight gain and hepatotoxicity are not common adverse effects.

12. An occupational health nurse is screening a group of workers for diabetes. What statement should the nurse interpret as suggestive of diabetes? A) Ive always been a fan of sweet foods, but lately Im turned off by them. B) Lately, I drink and drink and cant seem to quench my thirst. C) No matter how much sleep I get, it seems to take me hours to wake up. D) When I went to the washroom the last few days, my urine smelled odd.

B Feedback: Classic clinical manifestations of diabetes include the three Ps: polyuria, polydipsia, and polyphagia. Lack of interest in sweet foods, fatigue, and foul-smelling urine are not suggestive of diabetes.

23. A patient with type 2 diabetes achieves adequate glycemic control through diet and exercise. Upon being admitted to the hospital for a cholecystectomy, however, the patient has required insulin injections on two occasions. The nurse would identify what likely cause for this short-term change in treatment? A) Alterations in bile metabolism and release have likely caused hyperglycemia. B) Stress has likely caused an increase in the patients blood sugar levels. C) The patient has likely overestimated her ability to control her diabetes using nonpharmacologic measures. D) The patients volatile fluid balance surrounding surgery has likely caused unstable blood sugars.

B Feedback: During periods of physiologic stress, such as surgery, blood glucose levels tend to increase, because levels of stress hormones (epinephrine, norepinephrine, glucagon, cortisol, and growth hormone) increase. The patients need for insulin is unrelated to the action of bile, the patients overestimation of previous blood sugar control, or fluid imbalance.

39. A diabetic patient calls the clinic complaining of having a flu bug. The nurse tells him to take his regular dose of insulin. What else should the nurse tell the patient? A) Make sure to stick to your normal diet. B) Try to eat small amounts of carbs, if possible. C) Ensure that you check your blood glucose every hour. D) For now, check your urine for ketones every 8 hours.

B Feedback: For prevention of DKA related to illness, the patient should attempt to consume frequent small portions of carbohydrates (including foods usually avoided, such as juices, regular sodas, and gelatin). Drinking fluids every hour is important to prevent dehydration. Blood glucose and urine ketones must be assessed every 3 to 4 hours.

33. A patient with a history of type 1 diabetes has just been admitted to the critical care unit (CCU) for diabetic ketoacidosis. The CCU nurse should prioritize what assessment during the patients initial phase of treatment? A) Monitoring the patient for dysrhythmias B) Maintaining and monitoring the patients fluid balance C) Assessing the patients level of consciousness D) Assessing the patient for signs and symptoms of venous thromboembolism

B Feedback: In addition to treating hyperglycemia, management of DKA is aimed at correcting dehydration, electrolyte loss, and acidosis before correcting the hyperglycemia with insulin. The nurse should monitor the patient for dysrhythmias, decreased LOC and VTE, but restoration and maintenance of fluid balance is the highest priority.

15. While assisting with the surgical removal of an adrenal tumor, the OR nurse is aware that the patients vital signs may change upon manipulation of the tumor. What vital sign changes would the nurse expect to see? A) Hyperthermia and tachypnea B) Hypertension and heart rate changes C) Hypotension and hypothermia D) Hyperthermia and bradycardia

B Feedback: Manipulation of the tumor during surgical excision may cause release of stored epinephrine and norepinephrine, with marked increases in BP and changes in heart rate. The use of sodium nitroprusside and alpha-adrenergic blocking agents may be required during and after surgery. While other vital sign changes may occur related to surgical complications, the most common changes are related to hypertension and changes in the heart rate.

39. A patient who has been taking corticosteroids for several months has been experiencing muscle wasting. The patient has asked the nurse for suggestions to address this adverse effect. What should the nurse recommend? A) Activity limitation to conserve energy B) Consumption of a high-protein diet C) Use of OTC vitamin D and calcium supplements D) Passive range-of-motion exercises

B Feedback: Muscle wasting can be partly addressed through increased protein intake. Passive ROM exercises maintain flexibility, but do not build muscle mass. Vitamin D and calcium supplements do not decrease muscle wasting. Activity limitation would exacerbate the problem.

8. The nurse is discussing macrovascular complications of diabetes with a patient. The nurse would address what topic during this dialogue? A) The need for frequent eye examinations for patients with diabetes B) The fact that patients with diabetes have an elevated risk of myocardial infarction C) The relationship between kidney function and blood glucose levels D) The need to monitor urine for the presence of albumin

B Feedback: Myocardial infarction and stroke are considered macrovascular complications of diabetes, while the effects on vision and renal function are considered to be microvascular.

16. A nurse is caring for a patient with type 1 diabetes who is being discharged home tomorrow. What is the best way to assess the patients ability to prepare and self-administer insulin? A) Ask the patient to describe the process in detail. B) Observe the patient drawing up and administering the insulin. C) Provide a health education session reviewing the main points of insulin delivery. D) Review the patients first hemoglobin A1C result after discharge.

B Feedback: Nurses should assess the patients ability to perform diabetes related self-care as soon as possible during the hospitalization or office visit to determine whether the patient requires further diabetes teaching. While consulting a home care nurse is beneficial, an initial assessment should be performed during the hospitalization or office visit. Nurses should directly observe the patient performing the skills such as insulin preparation and infection, blood glucose monitoring, and foot care. Simply questioning the patient about these skills without actually observing performance of the skill is not sufficient. Further education does not guarantee learning.

5. A medical nurse is caring for a patient with type 1 diabetes. The patients medication administration record includes the administration of regular insulin three times daily. Knowing that the patients lunch tray will arrive at 11:45, when should the nurse administer the patients insulin? A) 10:45 B) 11:15 C) 11:45 D) 11:50

B Feedback: Regular insulin is usually administered 2030 min before a meal. Earlier administration creates a risk for hypoglycemia; later administration creates a risk for hyperglycemia.

36. A diabetes nurse is assessing a patients knowledge of self-care skills. What would be the most appropriate way for the educator to assess the patients knowledge of nutritional therapy in diabetes? A) Ask the patient to describe an optimally healthy meal. B) Ask the patient to keep a food diary and review it with the nurse. C) Ask the patients family what he typically eats. D) Ask the patient to describe a typical days food intake.

B Feedback: Reviewing the patients actual food intake is the most accurate method of gauging the patients diet.

6. A patient has just been diagnosed with type 2 diabetes. The physician has prescribed an oral antidiabetic agent that will inhibit the production of glucose by the liver and thereby aid in the control of blood glucose. What type of oral antidiabetic agent did the physician prescribe for this patient? A) A sulfonylurea B) A biguanide C) A thiazolidinedione D) An alpha glucosidase inhibitor

B Feedback: Sulfonylureas exert their primary action by directly stimulating the pancreas to secrete insulin and therefore require a functioning pancreas to be effective. Biguanides inhibit the production of glucose by the liver and are in used in type 2 diabetes to control blood glucose levels. Thiazolidinediones enhance insulin action at the receptor site without increasing insulin secretion from the beta cells of the pancreas. Alpha glucosidase inhibitors work by delaying the absorption of glucose in the intestinal system, resulting in a lower postprandial blood glucose level.

22. The physician has ordered a fluid deprivation test for a patient suspected of having diabetes insipidus. During the test, the nurse should prioritize what assessments? A) Temperature and oxygen saturation B) Heart rate and BP C) Breath sounds and bowel sounds D) Color, warmth, movement, and sensation of extremities

B Feedback: The fluid deprivation test is carried out by withholding fluids for 8 to 12 hours or until 3% to 5% of the body weight is lost. The patients condition needs to be monitored frequently during the test, and the test is terminated if tachycardia, excessive weight loss, or hypotension develops. Consequently, BP and heart rate monitoring are priorities over the other listed assessments.

17. An elderly patient comes to the clinic with her daughter. The patient is a diabetic and is concerned about foot care. The nurse goes over foot care with the patient and her daughter as the nurse realizes that foot care is extremely important. Why would the nurse feel that foot care is so important to this patient? A) An elderly patient with foot ulcers experiences severe foot pain due to the diabetic polyneuropathy. B) Avoiding foot ulcers may mean the difference between institutionalization and continued independent living. C) Hypoglycemia is linked with a risk for falls; this risk is elevated in older adults with diabetes. D) Oral antihyperglycemics have the possible adverse effect of decreased circulation to the lower extremities.

B Feedback: The nurse recognizes that providing information on the long-term complicationsespecially foot and eye problemsassociated with diabetes is important. Avoiding amputation through early detection of foot ulcers may mean the difference between institutionalization and continued independent living for the elderly person with diabetes. While the nurse recognizes that hypoglycemia is a dangerous situation and may lead to falls, hypoglycemia is not directly connected to the importance of foot care. Decrease in circulation is related to vascular changes and is not associated with drugs administered for diabetes.

36. What should the nurse teach a patient on corticosteroid therapy in order to reduce the patients risk of adrenal insufficiency? A) Take the medication late in the day to mimic the bodys natural rhythms. B) Always have enough medication on hand to avoid running out. C) Skip up to 2 doses in cases of illness involving nausea. D) Take up to 1 extra dose per day during times of stress.

B Feedback: The patient and family should be informed that acute adrenal insufficiency and underlying symptoms will recur if corticosteroid therapy is stopped abruptly without medical supervision. The patient should be instructed to have an adequate supply of the corticosteroid medication always available to avoid running out. Doses should not be skipped or added without explicit instructions to do so. Corticosteroids should normally be taken in the morning to mimic natural rhythms.

18. A patient with a diagnosis of syndrome of inappropriate antidiuretic hormone secretion (SIADH) is being cared for on the critical care unit. The priority nursing diagnosis for a patient with this condition is what? A) Risk for peripheral neurovascular dysfunction B) Excess fluid volume C) Hypothermia D) Ineffective airway clearance

B Feedback: The priority nursing diagnosis for a patient with SIADH is excess fluid volume, as the patient retains fluids and develops a sodium deficiency. Restricting fluid intake is a typical intervention for managing this syndrome. Temperature imbalances are not associated with SIADH. The patient is not at risk for neurovascular dysfunction or a compromised airway.

26. The nurses assessment of a patient with thyroidectomy suggests tetany and a review of the most recent blood work corroborate this finding. The nurse should prepare to administer what intervention? A) Oral calcium chloride and vitamin D B) IV calcium gluconate C) STAT levothyroxine D) Administration of parathyroid hormone (PTH)

B Feedback: When hypocalcemia and tetany occur after a thyroidectomy, the immediate treatment is administration of IV calcium gluconate. This has a much faster therapeutic effect than PO calcium or vitamin D supplements. PTH and levothyroxine are not used to treat this complication.

21. A patient with suspected adrenal insufficiency has been ordered an adrenocorticotropic hormone (ACTH) stimulation test. Administration of ACTH caused a marked increase in cortisol levels. How should the nurse interpret this finding? A) The patients pituitary function is compromised. B) The patients adrenal insufficiency is not treatable. C) The patient has insufficient hypothalamic function. D) The patient would benefit from surgery

A Feedback: An adrenal response to the administration of a stimulating hormone suggests inadequate production of the stimulating hormone. In this case, ACTH is produced by the pituitary and, consequently, pituitary hypofunction is suggested. Hypothalamic function is not relevant to the physiology of this problem. Treatment exists, although surgery is not likely indicated.

35. A patient with Cushing syndrome as a result of a pituitary tumor has been admitted for a transsphenoidal hypophysectomy. What would be most important for the nurse to monitor before, during, and after surgery? A) Blood glucose B) Assessment of urine for blood C) Weight D) Oral temperature

A Feedback: Before, during, and after this surgery, blood glucose monitoring and assessment of stools for blood are carried out. The patients blood sugar is more likely to be volatile than body weight or temperature. Hematuria is not a common complication.

10. A 15-year-old child is brought to the emergency department with symptoms of hyperglycemia and is subsequently diagnosed with diabetes. Based on the fact that the childs pancreatic beta cells are being destroyed, the patient would be diagnosed with what type of diabetes? A) Type 1 diabetes B) Type 2 diabetes C) Noninsulin-dependent diabetes D) Prediabetes

A Feedback: Beta cell destruction is the hallmark of type 1 diabetes. Noninsulin-dependent diabetes is synonymous with type 2 diabetes, which involves insulin resistance and impaired insulin secretion, but not beta cell destruction. Prediabetes is characterized by normal glucose metabolism, but a previous history of hyperglycemia, often during illness or pregnancy.

37. The most recent blood work of a patient with a longstanding diagnosis of type 1 diabetes has shown the presence of microalbuminuria. What is the nurses most appropriate action? A) Teach the patient about actions to slow the progression of nephropathy. B) Ensure that the patient receives a comprehensive assessment of liver function. C) Determine whether the patient has been using expired insulin. D) Administer a fluid challenge and have the test repeated.

A Feedback: Clinical nephropathy eventually develops in more than 85% of people with microalbuminuria. As such, educational interventions addressing this microvascular complication are warranted. Expired insulin does not cause nephropathy, and the patients liver function is not likely affected. There is no indication for the use of a fluid challenge.

2. A patient presents to the clinic complaining of symptoms that suggest diabetes. What criteria would support checking blood levels for the diagnosis of diabetes? A) Fasting plasma glucose greater than or equal to 126 mg/dL B) Random plasma glucose greater than 150 mg/dL C) Fasting plasma glucose greater than 116 mg/dL on 2 separate occasions D) Random plasma glucose greater than 126 mg/dL

A Feedback: Criteria for the diagnosis of diabetes include symptoms of diabetes plus random plasma glucose greater than or equal to 200 mg/dL, or a fasting plasma glucose greater than or equal to 126 mg/dL.

26. A patient with a longstanding diagnosis of type 1 diabetes has a history of poor glycemic control. The nurse recognizes the need to assess the patient for signs and symptoms of peripheral neuropathy. Peripheral neuropathy constitutes a risk for what nursing diagnosis? A) Infection B) Acute pain C) Acute confusion D) Impaired urinary elimination

A Feedback: Decreased sensations of pain and temperature place patients with neuropathy at increased risk for injury and undetected foot infections. The neurologic changes associated with peripheral neuropathy do not normally result in pain, confusion, or impairments in urinary function.

28. Following an addisonian crisis, a patients adrenal function has been gradually regained. The nurse should ensure that the patient knows about the need for supplementary glucocorticoid therapy in which of the following circumstances? A) Episodes of high psychosocial stress B) Periods of dehydration C) Episodes of physical exertion D) Administration of a vaccine

A Feedback: During stressful procedures or significant illnesses, additional supplementary therapy with glucocorticoids is required to prevent addisonian crisis. Physical activity, dehydration and vaccine administration would not normally be sufficiently demanding such to require glucocorticoids.

23. A nurse works in a walk-in clinic. The nurse recognizes that certain patients are at higher risk for different disorders than other patients. What patient is at a greater risk for the development of hypothyroidism? A) A 75-year-old female patient with osteoporosis B) A 50-year-old male patient who is obese C) A 45-year-old female patient who used oral contraceptives D) A 25-year-old male patient who uses recreational drugs

A Feedback: Even though osteoporosis is not a risk factor for hypothyroidism, the condition occurs most frequently in older women.

30. A nurse is conducting a class on how to self-manage insulin regimens. A patient asks how long a vial of insulin can be stored at room temperature before it goes bad. What would be the nurses best answer? A) If you are going to use up the vial within 1 month it can be kept at room temperature. B) If a vial of insulin will be used up within 21 days, it may be kept at room temperature. C) If a vial of insulin will be used up within 2 weeks, it may be kept at room temperature. D) If a vial of insulin will be used up within 1 week, it may be kept at room temperature.

A Feedback: If a vial of insulin will be used up within 1 month, it may be kept at room temperature.

27. A patient has been brought to the emergency department by paramedics after being found unconscious. The patients Medic Alert bracelet indicates that the patient has type 1 diabetes and the patients blood glucose is 22 mg/dL (1.2 mmol/L). The nurse should anticipate what intervention? A) IV administration of 50% dextrose in water B) Subcutaneous administration of 10 units of Humalog C) Subcutaneous administration of 12 to 15 units of regular insulin D) IV bolus of 5% dextrose in 0.45% NaCl

A Feedback: In hospitals and emergency departments, for patients who are unconscious or cannot swallow, 25 to 50 mL of 50% dextrose in water (D50W) may be administered IV for the treatment of hypoglycemia. Five percent dextrose would be inadequate and insulin would exacerbate the patients condition.

34. A patient has been living with type 2 diabetes for several years, and the nurse realizes that the patient is likely to have minimal contact with the health care system. In order to ensure that the patient maintains adequate blood sugar control over the long term, the nurse should recommend which of the following? A) Participation in a support group for persons with diabetes B) Regular consultation of websites that address diabetes management C) Weekly telephone check-ins with an endocrinologist D) Participation in clinical trials relating to antihyperglycemics

A Feedback: Participation in support groups is encouraged for patients who have had diabetes for many years as well as for those who are newly diagnosed. This is more interactive and instructive than simply consulting websites. Weekly telephone contact with an endocrinologist is not realistic in most cases. Participation in research trials may or may not be beneficial and appropriate, depending on patients circumstances.

1. The nurse is caring for a patient diagnosed with hypothyroidism secondary to Hashimotos thyroiditis. When assessing this patient, what sign or symptom would the nurse expect? A)Fatigue B) Bulging eyes C) Palpitations D) Flushed skin

A Feedback: Symptoms of hypothyroidism include extreme fatigue, hair loss, brittle nails, dry skin, voice huskiness or hoarseness, menstrual disturbance, and numbness and tingling of the fingers. Bulging eyes, palpitations, and flushed skin would be signs and symptoms of hyperthyroidism.

28. A diabetic nurse is working for the summer at a camp for adolescents with diabetes. When providing information on the prevention and management of hypoglycemia, what action should the nurse promote? A) Always carry a form of fast-acting sugar. B) Perform exercise prior to eating whenever possible. C) Eat a meal or snack every 8 hours. D) Check blood sugar at least every 24 hours.

A Feedback: The following teaching points should be included in information provided to the patient on how to prevent hypoglycemia: Always carry a form of fast-acting sugar, increase food prior to exercise, eat a meal or snack every 4 to 5 hours, and check blood sugar regularly.

7. A diabetes nurse educator is teaching a group of patients with type 1 diabetes about sick day rules. What guideline applies to periods of illness in a diabetic patient? A) Do not eliminate insulin when nauseated and vomiting. B) Report elevated glucose levels greater than 150 mg/dL. C) Eat three substantial meals a day, if possible. D) Reduce food intake and insulin doses in times of illness.

A Feedback: The most important issue to teach patients with diabetes who become ill is not to eliminate insulin doses when nausea and vomiting occur. Rather, they should take their usual insulin or oral hypoglycemic agent dose, then attempt to consume frequent, small portions of carbohydrates. In general, blood sugar levels will rise but should be reported if they are greater than 300 mg/dL.

18. A diabetic educator is discussing sick day rules with a newly diagnosed type 1 diabetic. The educator is aware that the patient will require further teaching when the patient states what? A) I will not take my insulin on the days when I am sick, but I will certainly check my blood sugar every 2 hours. B) If I cannot eat a meal, I will eat a soft food such as soup, gelatin, or pudding six to eight times a day. C) I will call the doctor if I am not able to keep liquids in my body due to vomiting or diarrhea. D) I will call the doctor if my blood sugar is over 300 mg/dL or if I have ketones in my urine.

A Feedback: The nurse must explanation the sick day rules again to the patient who plans to stop taking insulin when sick. The nurse should emphasize that the patient should take insulin agents as usual and test ones blood sugar and urine ketones every 3 to 4 hours. In fact, insulin-requiring patients may need supplemental doses of regular insulin every 3 to 4 hours. The patient should report elevated glucose levels (greater than 300 mg/dL or as otherwise instructed) or urine ketones to the physician. If the patient is not able to eat normally, the patient should be instructed to substitute soft foods such a gelatin, soup, and pudding. If vomiting, diarrhea, or fever persists, the patient should have an intake of liquids every 30 to 60 minutes to prevent dehydration.

32. The nurse providing care for a patient with Cushing syndrome has identified the nursing diagnosis of risk for injury related to weakness. How should the nurse best reduce this risk? A) Establish falls prevention measures. B) Encourage bed rest whenever possible. C) Encourage the use of assistive devices. D) Provide constant supervision.

A Feedback: The nurse should take action to prevent the patients risk for falls. Bed rest carries too many harmful effects, however, and assistive devices may or may not be necessary. Constant supervision is not normally required or practicable.

10. You are developing a care plan for a patient with Cushing syndrome. What nursing diagnosis would have the highest priority in this care plan? A) Risk for injury related to weakness B) Ineffective breathing pattern related to muscle weakness C) Risk for loneliness related to disturbed body image D) Autonomic dysreflexia related to neurologic changes

A Feedback: The nursing priority is to decrease the risk of injury by establishing a protective environment. The patient who is weak may require assistance from the nurse in ambulating to prevent falls or bumping corners or furniture. The patients breathing will not be affected and autonomic dysreflexia is not a plausible risk. Loneliness may or may not be an issue for the patient, but safety is a priority.

13. A patient presents at the walk-in clinic complaining of diarrhea and vomiting. The patient has a documented history of adrenal insufficiency. Considering the patients history and current symptoms, the nurse should anticipate that the patient will be instructed to do which of the following? A) Increase his intake of sodium until the GI symptoms improve. B) Increase his intake of potassium until the GI symptoms improve. C) Increase his intake of glucose until the GI symptoms improve. D) Increase his intake of calcium until the GI symptoms improve.

A Feedback: The patient will need to supplement dietary intake with added salt during episodes of GI losses of fluid through vomiting and diarrhea to prevent the onset of addisonian crisis. While the patient may experience the loss of other electrolytes, the major concern is the replacement of lost sodium.

31. A patient is undergoing testing for suspected adrenocortical insufficiency. The care team should ensure that the patient has been assessed for the most common cause of adrenocortical insufficiency. What is the most common cause of this health problem? A) Therapeutic use of corticosteroids B) Pheochromocytoma C) Inadequate secretion of ACTH D) Adrenal tumor

A Feedback: Therapeutic use of corticosteroids is the most common cause of adrenocortical insufficiency. The other options also cause adrenocortical insufficiency, but they are not the most common causes.

22. A nurse is caring for a patient newly diagnosed with type 1 diabetes. The nurse is educating the patient about self-administration of insulin in the home setting. The nurse should teach the patient to do which of the following? A) Avoid using the same injection site more than once in 2 to 3 weeks. B) Avoid mixing more than one type of insulin in a syringe. C) Cleanse the injection site thoroughly with alcohol prior to injecting. D) Inject at a 45 angle.

A Feedback: To prevent lipodystrophy, the patient should try not to use the same site more than once in 2 to 3 weeks. Mixing different types of insulin in a syringe is acceptable, within specific guidelines, and the needle is usually inserted at a 90 angle. Cleansing the injection site with alcohol is optional.

33. A patient with Cushing syndrome has been hospitalized after a fall. The dietician consulted works with the patient to improve the patients nutritional intake. What foods should a patient with Cushing syndrome eat to optimize health? Select all that apply. A) Foods high in vitamin D B) Foods high in calories C) Foods high in protein D) Foods high in calcium E) Foods high in sodium

A, C, D Feedback: Foods high in vitamin D, protein, and calcium are recommended to minimize muscle wasting and osteoporosis. Referral to a dietitian may assist the patient in selecting appropriate foods that are also low in sodium and calories.

11. The nurse is performing a shift assessment of a patient with aldosteronism. What assessments should the nurse include? Select all that apply. A) Urine output B) Signs or symptoms of venous thromboembolism C) Peripheral pulses D) Blood pressure E) Skin integrity

A, D Feedback: The principal action of aldosterone is to conserve body sodium. Alterations in aldosterone levels consequently affect urine output and BP. The patients peripheral pulses, risk of VTE, and skin integrity are not typically affected by aldosteronism.

11. While assessing a client with Graves disease, the nurse notes that the client's temperature has risen 1° F (1° C). What does the nurse do first? a. Turn the lights down and shut the patient's door. b. Call for an immediate electrocardiogram (ECG). c. Calculate the client's apical-radial pulse deficit. d. Administer a dose of acetaminophen.

ANS: A A temperature increase of 1° F (5/9° C) may indicate the development of thyroid storm, and the primary health care provider or RRT needs to be notified. But before notifying the provider, the nurse should first take measures to reduce environmental stimuli that increase the risk of cardiac complications. The nurse can then call for an ECG. The apical-radial pulse deficit would not be necessary, and acetaminophen is not needed because the temperature increase is due to thyroid activity.

2. A client is scheduled to have a glycosylated hemoglobin (A1C) drawn and asks the nurse why she has to have it. How would the nurse respond? a. "It measures your average blood glucose level for the past 3 months." b. "It determines what type of anemia you may have." c. "It measures the amount of liver glycogen you have." d. "It determines you have some type of leukemia or other blood cancer."

ANS: A A1C measures the average blood glucose level to determine if the client is a diabetic or how controlled a diabetic client is.

6. A nurse teaches a client who is diagnosed with diabetes mellitus. Which statement would the nurse include in this client's plan of care to delay the onset of microvascular and macrovascular complications? a. "Maintain tight glycemic control and prevent hyperglycemia." b. "Restrict your fluid intake to no more than 2 L a day." c. "Prevent hypoglycemia by eating a bedtime snack." d. "Limit your intake of protein to prevent ketoacidosis."

ANS: A Hyperglycemia is a critical factor in the pathogenesis of long-term diabetic complications. Maintaining tight glycemic control will help delay the onset of complications. Restricting fluid intake is not part of the treatment plan for patients with diabetes. Preventing hypoglycemia and ketosis, although important, is not as important as maintaining daily glycemic control.

18. The nurse assesses a client with diabetic ketoacidosis. Which assessment finding would the nurse correlate with this condition? a. Increased rate and depth of respiration b. Extremity tremors followed by seizure activity c. Oral temperature of 102° F (38.9° C) d. Severe orthostatic hypotension

ANS: A Ketoacidosis decreases the pH of the blood, stimulating the respiratory control areas of the brain to buffer the effects of increasing acidosis. The rate and depth of respiration are increased (Kussmaul respirations) in an attempt to excrete more acids by exhalation. Tremors, elevated temperature, and orthostatic hypotension are not associated with ketoacidosis.

5. A nurse cares for a client who has a family history of diabetes mellitus. The client states, "My father has type 1 diabetes mellitus. Will I develop this disease as well?" How would the nurse respond? a. "Your risk of diabetes is higher than the general population, but it may not occur." b. "No genetic risk is associated with the development of type 1 diabetes mellitus." c. "The risk for becoming a diabetic is 50% because of how it is inherited." d. "Female children do not inherit diabetes mellitus, but male children will."

ANS: A Risk for type 1 diabetes is determined by inheritance of genes coding for HLA-DR and HLA-DQ tissue types. Clients who have one parent with type 1 diabetes are at increased risk for its development. Diabetes (type 1) seems to require interaction between inherited risk and environmental factors, so not everyone with these genes develops diabetes. The other statements are not accurate.

4. A nurse assesses a client who is prescribed a medication that stimulates beta1 receptors. Which assessment finding would indicate that the medication is effective? a. Heart rate of 92 beats/min b. Respiratory rate of 18 breaths/min c. Oxygenation saturation of 92% d. Blood pressure of 144/69 mm Hg

ANS: A Stimulation of beta1 receptor sites in the heart has positive chronotropic and inotropic actions. The nurse expects an increase in heart rate and increased cardiac output. The other vital signs are within normal limits and do not indicate any response to the medication.

3. The nurse is caring for a client who has acromegaly. What physical change would the nurse expect to observe? a. Large hands and face b. Thin, dry skin c. Short height d. Truncal obesity

ANS: A The client who has acromegaly has an excess of growth hormone as an adult and therefore has a large musculoskeletal structure that is readily observed.

11. A nurse is reviewing care for a client who has syndrome of inappropriate antidiuretic hormone (SIADH) with assistive personnel. What statement by the AP indicates understanding of this client's care? a. "I will weigh the client carefully before breakfast and compare with yesterday's weight." b. "I will encourage plenty of fluids to promote urination and prevent dehydration." c. "I will teach the client not to select high-sodium or salty foods on the menu." d. "I will assess the client's mucous membranes and skin for signs of dehydration."

ANS: A The client with SIADH usually has a fluid restriction, not an increase in fluids. It is the role of the RN rather than AP to perform assessments and provide health teaching. The AP needs to weigh the client daily and report a significant weight changes.

5. A nurse collaborates with assistive personnel (AP) to provide care for a client who is prescribed a 24-hour urine specimen collection. Which statement would the nurse include when teaching the AP about this activity? a. "Note the time of the client's first void and collect urine for 24 hours." b. "Add the preservative to the container at the end of the test." c. "Start the collection by saving the first urine of the morning." d. "It is okay if one urine sample during the 24 hours is not collected."

ANS: A The collection of a 24-hour urine specimen is often delegated to AP. The nurse must ensure that the AP understands the proper process for collecting the urine. The 24-hour urine collection specimen is started after the client's first urination. The first urine specimen is discarded because there is no way to know how long it has been in the bladder, but the time of the client's first void is noted. The client adds all urine voided after that first discarded specimen during the next 24 hours. When the 24-hour mark is reached, the client voids one last time and adds this specimen to the collection. The preservative, if used, must be added to the container at the beginning of the collection. All urine samples need to be collected for the test results to be accurate.

11. A nurse cares for a patient who is prescribed pioglitazone. After 6 months of therapy, the client reports that he has a new onset of ankle edema. What assessment question would the nurse take? a. "Have you gained unexpected weight this week?" b. "Has your urinary output declined recently?" c. "Have you had fever and achiness this week?" d. "Have you had abdominal pain recently?"

ANS: A Thiazolidinediones (including pioglitazone) can cause cardiovascular adverse effects including health failure which is manifested by peripheral edema and unintentional weight gain. The client should have been taught to weigh every week and report sudden increases in weight.

26. A nurse assesses a client who has diabetes mellitus and notes that the client is awake and alert, but shaky, diaphoretic, and weak. Five minutes after administering a half-cup (120 mL) of orange juice, the client's signs and symptoms have not changed. What action would the nurse take next? a. Administer another half-cup (120 mL) of orange juice. b. Administer a half-ampule of dextrose 50% intravenously. c. Administer 10 units of regular insulin subcutaneously. d. Administer 1 mg of glucagon intramuscularly.

ANS: A This patient is experiencing mild hypoglycemia. For mild hypoglycemic manifestations, the nurse would administer oral glucose in the form of orange juice. If the symptoms do not resolve immediately, the treatment would be repeated. The patient does not need intravenous dextrose, insulin, or glucagon.

3. A nurse assesses clients who have endocrine disorders. Which assessment findings are paired correctly with the endocrine disorder? (Select all that apply.) a. Excessive thyroid-stimulating hormone—increased bone formation b. Excessive melanocyte-stimulating hormone—darkening of the skin c. Excessive parathyroid hormone—synthesis and release of corticosteroids d. Excessive antidiuretic hormone—increased urinary output e. Excessive adrenocorticotropic hormone—increased bone resorption

ANS: A,B Thyroid-stimulating hormone targets thyroid tissue and stimulates the formation of bone. Melanocyte-stimulating hormone stimulates melanocytes and promotes pigmentation or the darkening of the skin. Parathyroid hormone stimulates bone resorption. Antidiuretic hormone targets the kidney and promotes water reabsorption, causing a decrease in urinary output. Adrenocorticotropic hormone targets the adrenal cortex and stimulates the synthesis and release of corticosteroids.

4. A nurse provides diabetic education at a public health fair. Which disorders would the nurse include as complications of diabetes mellitus? (Select all that apply.) a. Stroke b. Kidney failure c. Blindness d. Respiratory failure e. Cirrhosis

ANS: A,B,C Complications of diabetes mellitus are caused by macrovascular and microvascular changes. Macrovascular complications include coronary artery disease, cerebrovascular disease, and peripheral vascular disease. Microvascular complications include nephropathy, retinopathy, and neuropathy. Respiratory failure and cirrhosis are not complications of diabetes mellitus.

1. A nurse cares for clients with hormone disorders. Which are common key features of hormones? (Select all that apply.) a. Hormones may travel long distances to get to their target tissues. b. Continued hormone activity requires continued production and secretion. c. Control of hormone activity is caused by negative feedback mechanisms. d. Most hormones are stored in the target tissues for use later. e. Most hormones cause target tissues to change activities by changing gene activity.

ANS: A,B,C Hormones are secreted by endocrine glands and travel through the body to reach their target tissues. Hormone activity can increase or decrease according to the body's needs, and continued hormone activity requires continued production and secretion. Control is maintained via negative feedback. Hormones are not stored for later use, and they do not alter genetic activity.

6. The nurse is caring for a client who has severe hypoglycemia and is experiencing a seizure. What actions will the nurse take at this time? (Select all that apply.) a. Administer glucagon 1 mg subcutaneously. b. Be sure the bed side rails are in the up position. c. Notify the primary health care provider immediately. d. Monitor the client's blood glucose level. e. Increase the intravenous infusion rate immediately.

ANS: A,B,C,D The client who has severe hypoglycemia often has a blood sugar of less than 20 mg/dL (1.0 mmol/L) and may be unconscious or seizing. Therefore, the client cannot swallow and needs glucagon. To keep the client safe during the seizure, the nurse ensures that the side rails are up to prevent the client from falling out of bed. The nurse would also monitor the client's blood sugar to evaluate the effectiveness of the interventions.

1. The nurse is caring for a client who has possible hypothyroidism. What possible risk factors can cause this health problem? (Select all that apply.) a. Lithium drug therapy b. Thyroid cancer c. Autoimmune thyroid disease d. Iodine deficiency e. Laryngitis f. Pituitary tumors

ANS: A,B,C,D,F All of these factors place a client at risk for hypothyroidism except for laryngitis which is an inflammation of the larynx.

5. A nurse collaborates with the interprofessional team to develop a plan of care for a client who is newly diagnosed with diabetes mellitus. Which team members would the nurse include in this interprofessional team meeting? (Select all that apply.) a. Registered dietitian nutritionist b. Clinical pharmacist c. Occupational therapist d. Primary health care provider e. Speech-language pathologist

ANS: A,B,D When planning care for a client newly diagnosed with diabetes mellitus, the nurse would collaborate with a registered dietitian nutritionist, clinical pharmacist, and primary health care provider. The focus of treatment for a newly diagnosed client would be nutrition, medication therapy, and education. The nurse could also consult with a diabetic educator. There is no need for occupational therapy or speech therapy at this time.

1. A client is admitted with a possible diagnosis of diabetes insipidus (DI). What assessment findings would the nurse expect? (Select all that apply.) a. Hypotension b. Increased urinary output c. Concentrated urine d. Decreased thirst e. Poor skin turgor f. Bradycardia

ANS: A,B,E The client who has DI has excessive urination and dehydration. Clients who are dehydrated have decreased blood pressure, increased pulse (tachycardia), and poor skin turgor. The urine is dilute with a low specific gravity.

2. A nurse assesses a patient who is experiencing diabetic ketoacidosis (DKA). For which assessment findings would the nurse monitor the client? (Select all that apply.) a. Deep and fast respirations b. Decreased urine output c. Tachycardia d. Dependent pulmonary crackles e. Orthostatic hypotension

ANS: A,C,E DKA leads to dehydration, which is manifested by tachycardia and orthostatic hypotension. Usually, patients have Kussmaul respirations, which are fast and deep. Increased urinary output (polyuria) is severe. Because of diuresis and dehydration, peripheral edema and crackles do not occur.

3. A nurse teaches a client with hyperthyroidism. Which dietary modifications should the nurse include in this client's health teaching? (Select all that apply.) a. Increased carbohydrates b. Decreased fats c. Increased calorie intake d. Supplemental vitamins e. Increased proteins

ANS: A,C,E The client is hypermetabolic and has an increased need for carbohydrates, calories, and proteins. Proteins are especially important because the client is at risk for a negative nitrogen balance. There is no need to decrease fat intake or take supplemental vitamins.

2. A nurse cares for a client with a hypofunctioning anterior pituitary gland. Which hormones would the nurse expect to be decreased as a result? (Select all that apply.) a. Thyroid-stimulating hormone b. Vasopressin c. Follicle-stimulating hormone d. Calcitonin e. Growth hormone

ANS: A,C,E Thyroid-stimulating hormone, follicle-stimulating hormone, and growth hormone all are secreted by the anterior pituitary gland. Vasopressin is secreted from the posterior pituitary gland. Calcitonin is secreted from the thyroid gland.

4. When caring for an older client who has hypothyroidism, what assessment findings will the nurse expect? (Select all that apply.) a. Lethargy b. Diarrhea c. Low body temperature d. Tachycardia e. Slowed speech f. Weight gain

ANS: A,C,E,F A client who has an underactive thyroid gland has a decreased metabolic rate, resulting in lethargy and lack of energy, weight gain, slowed speech, and decreased vital signs like a lowered body temperature. The client also typically has constipation (instead of diarrhea) due to slower peristalsis and bradycardia (instead of tachycardia).

4. A nurse assesses a client with Cushing disease. Which assessment findings would the nurse expect? (Select all that apply.) a. Moon face b. Weight loss c. Hypotension d. Petechiae e. Muscle atrophy

ANS: A,D,E Clinical manifestations of Cushing disease include moon face, weight gain, hypertension, petechiae, and muscle atrophy.

1. A nurse assesses adults at a health fair. Which adults would the nurse counsel to be tested for diabetes? (Select all that apply.) a. A 56-year-old African-American male b. A 22-year-old female with a 30-lb (13.6 kg) weight gain during pregnancy c. A 60-year-old male with a history of liver trauma d. A 48-year-old female with a sedentary lifestyle e. A 50-year-old male with a body mass index greater than 25 kg/m2 f. A 28-year-old female who gave birth to a baby weighing 9.2 lb (4.2 kg)

ANS: A,D,E,F Risk factors for type 2 diabetes include certain ethnic/racial groups (African Americans, American Indians, and Hispanics), obesity and physical inactivity, and giving birth to large babies. Liver trauma and a 30-lb (13.6 kg) gestational weight gain are not risk factors.

6. After teaching a client who is recovering from an endoscopic transsphenoidal hypophysectomy, the nurse assesses the client's understanding. Which statement made by the client indicates a correct understanding of the teaching? a. "I will wear dark glasses to prevent sun exposure." b. "I'll keep food on upper shelves so I do not have to bend over." c. "I must wash the incision with saline and redress it daily." d. "I should cough and deep breathe every 2 hours while I am awake."

ANS: B After this surgery, the client must take care to avoid activities that can increase intracranial pressure. The client should avoid bending from the waist and should not bear down, cough, or lie flat. With this approach, there is no incision to clean and dress. Protection from sun exposure is not necessary after this procedure.

10. A nurse teaches an older woman who has a decreased production of estrogen. Which statement would the nurse include in this client's teaching to decrease injury? a. "Drink at least 2 quarts (2 L) of fluids each day." b. "Walk around the neighborhood for daily exercise." c. "Bathe your perineal area twice a day." d. "You should check your blood glucose before meals."

ANS: B An older female with decreased production of estrogen is at risk for decreased bone density and fractures. The nurse would encourage the client to participate in weight-bearing exercises such as walking. Drinking fluids and performing perineal care will decrease vaginal drying but not injury. Older adults often have a decreased glucose tolerance, but this is not related to a decrease in estrogen.

1. A nurse is teaching a client with diabetes mellitus who asks, "Why is it necessary to maintain my blood glucose levels no lower than about 60 mg/dL (3.3 mmol/L)?" How would the nurse respond? a. "Glucose is the only fuel used by the body to produce the energy that it needs." b. "Your brain needs a constant supply of glucose because it cannot store it." c. "Without a minimum level of glucose, your body does not make red blood cells." d. "Glucose in the blood prevents the formation of lactic acid and prevents acidosis."

ANS: B Because the brain cannot synthesize or store significant amounts of glucose, a continuous supply from the body's circulation is needed to meet the fuel demands of the central nervous system. The nurse would want to educate the patient to prevent hypoglycemia. The body can use other sources of fuel, including fat and protein, and glucose is not involved in the production of red blood cells. Glucose in the blood will encourage glucose metabolism but is not directly responsible for lactic acid formation.

7. A nurse is caring for a patient who has excessive catecholamine release. Which assessment finding would the nurse correlate with this condition? a. Decreased blood pressure b. Increased pulse c. Decreased respiratory rate d. Increased urine output

ANS: B Catecholamines are responsible for the fight-or-flight stress response. Activation of the sympathetic nervous system can be correlated with tachycardia. Catecholamines do not decrease blood pressure or respiratory rate, nor do they increase urine output.

1. A nurse assesses clients for potential endocrine dysfunction. Which client is at greatest risk for a deficiency of gonadotropin and growth hormone? a. A 36-year-old female who has used oral contraceptives for 5 years b. A 42-year-old male who experienced head trauma 3 years ago c. A 55-year-old female with a severe allergy to shellfish and iodine d. A 64-year-old male with adult-onset diabetes mellitus

ANS: B Gonadotropin and growth hormone are anterior pituitary hormones. Head trauma is a common cause of anterior pituitary hypofunction. The other factors do not increase the risk of this condition.

6. A nurse assesses a female client who presents with hirsutism. Which question would the nurse ask when assessing this client? a. "How do you plan to pay for your treatments?" b. "How do you feel about yourself?" c. "What medications are you prescribed?" d. "What are you doing to prevent this from happening?"

ANS: B Hirsutism, or excessive hair growth on the face and body, can result from endocrine disorders. This may cause a disruption in body image, especially for female clients. The nurse would inquire into the client's body image and self-perception. Asking about the client's financial status or current medications does not address the client's immediate problem. The client is not doing anything to herself to cause the problem, nor can the client prevent it from happening.

10. A nurse cares for a client with adrenal hyperfunction. The client screams at her husband, bursts into tears, and throws her water pitcher against the wall. She then tells the nurse, "I feel like I am going crazy." How would the nurse respond? a. "I will ask your doctor to order a mental health consult for you." b. "You feel this way because of your hormone levels." c. "Can I bring you information about support groups?" d. "I will close the door to your room and restrict visitors."

ANS: B Hypercortisolism can cause the client to have neurotic or psychotic behaviors. The client needs to know that these behavior changes do not reflect a true mental or behavioral health disorder and will resolve when therapy results in lower and steadier blood cortisol levels. The client needs to understand this effect and does not need a psychiatrist, support groups, or restricted visitors at this time.

1. The nurse is planning health teaching for a client starting on levothyroxine. What health teaching about this drug would the nurse include? a. The need to take the drug when the client feels fatigued and weak. b. The need to report chest pain and dyspnea when starting the drug. c. The need to check blood pressure and pulse every day. d. The need to rotate injection sites when giving self the drug.

ANS: B Levothyroxine is a replacement hormone for clients who have hypothyroidism and is taken orally for life. Vital signs do not have to be checked every day, but the client should report any chest pain and dyspnea when first starting the drug.

9. A nurse teaches a client with type 2 diabetes mellitus who is prescribed glipizide (Glucotrol). Which statement would the nurse include in this client's teaching? a. "Change positions slowly when you get out of bed." b. "Avoid taking nonsteroidal anti-inflammatory drugs (NSAIDs)." c. "If you miss a dose of this drug, you can double the next dose." d. "Discontinue the medication if you develop a urinary infection."

ANS: B NSAIDs potentiate the hypoglycemic effects of sulfonylurea agents. Glipizide is a sulfonylurea. The other statements are not applicable to glipizide.

29. The nurse is caring for a client who has diabetes mellitus. The nurse administers 6 units of regular insulin and 10 units of NPH insulin at 7:00 a.m. (0700). At which time would the nurse assess the client for potential hypoglycemia related to the NPH insulin? a. 8:00 a.m. (0800) b. 4:00 p.m. (1600) c. 8:00 p.m. (2000) d. 11:00 p.m. (2300)

ANS: B Neutral protamine Hagedorn (NPH) is an intermediate-acting insulin with an onset of 1.5 hours, peak of 4 to 12 hours, and duration of action of 22 hours. Checking the client at 0800 would be too soon. Checking the patient at 2000 and 2300 would be too late. The nurse would check the patient at 1600 (4:00 p.m.).

24. A nurse assesses a client with diabetes mellitus. Which assessment finding would alert the nurse to decreased kidney function in this client? a. Urine specific gravity of 1.033 b. Presence of protein in the urine c. Elevated capillary blood glucose level d. Presence of ketone bodies in the urine

ANS: B Renal dysfunction often occurs in the client with diabetes. Proteinuria is a result of renal dysfunction. Specific gravity is elevated with dehydration. Elevated capillary blood glucose levels and ketones in the urine are consistent with diabetes mellitus but are not specific to renal function.

25. A nurse develops a dietary plan for a client with diabetes mellitus and new-onset microalbuminuria. Which component of the client's diet would the nurse decrease? a. Carbohydrates b. Proteins c. Fats d. Total calories

ANS: B Restriction of dietary protein is recommended for clients with microalbuminuria to delay progression to renal failure. The client's diet does not need to be decreased in carbohydrates, fats, or total calories.

8. A nurse teaches a patient about self-monitoring of blood glucose levels. Which statement would the nurse include in this client's teaching to prevent bloodborne infections? a. "Wash your hands after completing each test." b. "Do not share your monitoring equipment." c. "Blot excess blood from the strip with a cotton ball." d. "Use gloves when monitoring your blood glucose."

ANS: B Small particles of blood can adhere to the monitoring device, and infection can be transported from one user to another. Hepatitis B in particular can survive in a dried state for about a week. The client would be taught to avoid sharing any equipment, including the lancet holder. The client would also be taught to wash his or her hands before testing. He or she would not need to blot excess blood away from the strip or wear gloves.

34. A nurse reviews the chart and new prescriptions for a client with diabetic ketoacidosis: Vital Signs and Assessment Laboratory Results Medications Blood pressure: 90/62 mm Hg Pulse: 120 beats/min Respiratory rate: 28 breaths/min Urine output: 20 mL/hr via catheter Serum potassium: 2.6 mEq/L (2.6 mmol/L) Potassium chloride 40 mEq/L (40 mmol/L) IV bolus STAT Increase IV fluid to 100 mL/hr What action would the nurse take? a. Administer the potassium and then consult with the primary health care provider about the fluid prescription. b. Increase the intravenous rate and then consult with the primary health care provider about the potassium prescription. c. Administer the potassium first before increasing the infusion flow rate for the client. d. Increase the intravenous flow rate before administering the potassium to the client.

ANS: B The client is acutely ill and is severely dehydrated and hypokalemic, requiring more IV fluids and potassium. However, potassium would not be infused unless the urine output is at least 30 mL/hr. The nurse would first increase the IV rate and then consult with the primary health care provider about the potassium.

5. A nurse cares for a client who presents with bradycardia secondary to hypothyroidism. Which medication does the nurse prepare to administer? a. Atropine sulfate b. Levothyroxine c. Propranolol d. Epinephrine

ANS: B The treatment for bradycardia from hypothyroidism is to treat the hypothyroidism using levothyroxine. If the heart rate were so slow that it became an emergency, then atropine or epinephrine might be an option for short-term management. Propranolol is a beta blocker and would be contraindicated for a client with bradycardia.

28. A nurse teaches a client with diabetes mellitus about sick-day management. Which statement would the nurse include in this client's teaching? a. "When ill, avoid eating or drinking to reduce vomiting and diarrhea." b. "Monitor your blood glucose levels at least every 4 hours while sick." c. "If vomiting, do not use insulin or take your oral antidiabetic agent." d. "Try to continue your prescribed exercise regimen even if you are sick."

ANS: B When ill, the client should monGitRorAhDisEoSrLhAerBb.loCoOdMglucose at least every 4 hours. The client should continue taking the medication regimen while ill. The client should continue to eat and drink as tolerated but should not exercise while sick.

19. A nurse assesses a client who has diabetes mellitus. Which arterial blood gas values would the nurse identify as potential ketoacidosis in this client? a. pH 7.38, HCO3 22 mEq/L (22 mmol/L), PCO2 38 mm Hg, PO2 98 mm Hg b. pH 7.28, HCO3 18 mEq/L (18 mmol/L), PCO2 28 mm Hg, PO2 98 mm Hg c. pH 7.48, HCO3 28 mEq/L (28 mmol/L), PCO2 38 mm Hg, PO2 98 mm Hg d. pH 7.32, HCO3 22 mEq/L (22 mmol/L), PCO2 58 mm Hg, PO2 88 mm Hg

ANS: B When the lungs can no longer offset acidosis, the pH decreases to below normal. A client who has diabetic ketoacidosis would present with arterial blood gas values that show primary metabolic acidosis with decreased bicarbonate levels and a compensatory respiratory alkalosis with decreased carbon dioxide levels.

7. A nurse cares for a client who possibly has syndrome of inappropriate antidiuretic hormone (SIADH). The client's serum sodium level is 114 mEq/L (114 mmol/L). What nursing action would be appropriate? a. Consult with the dietitian about increased dietary sodium. b. Restrict the client's fluid intake to 600 mL/day. c. Handle the client gently by using turn sheets for repositioning. d. Instruct assistive personnel to measure intake and output.

ANS: B With SIADH, clients often have dilutional hyponatremia. The client needs a fluid restriction, sometimes to as little as 500 to 600 mL/24 hr. Adding sodium to the client's diet will not help if he or she is retaining fluid and diluting the sodium. The client is not at increased risk for fracture, so gentle handling is not an issue. The client would be on intake and output; however, this will monitor only the client's intake, so it is not the best answer. Reducing fluid intake will help increase the client's sodium.

3. A nurse teaches a client with Cushing disease. Which dietary requirements would the nurse include in this client's health teaching? (Select all that apply.) a. Low calcium b. Low carbohydrate c. Low protein d. Low calories e. Low sodium

ANS: B,D,E The client with Cushing disease has weight gain, muscle loss, hyperglycemia, and sodium retention. Dietary modifications need to include reduction of carbohydrates and total calories to prevent or reduce the degree of hyperglycemia. Sodium retention causes water retention and hypertension. Clients are encouraged to restrict their sodium intake moderately. Clients often have bone density loss and need more calcium. Increased protein intake will help decrease muscle loss.

7. The nurse is caring for a client who has diabetes mellitus type 1 and is experiencing hypoglycemia. Which assessment findings will the nurse expect? (Select all that apply.) a. Warm, dry skin b. Nervousness c. Rapid deep respirations d. Dehydration e. Ketoacidosis f. Blurred vision

ANS: B,F The client who has hypoglycemia is often anxious, nervous, and possibly confused. Due to lack of glucose, vision may be blurred or the client may report diplopia (double vision). Clients who have hyperglycemia from diabetes mellitus type 1 have warm skin, Kussmaul respirations that are rapid and deep, dehydration due to elevated blood glucose, and ketoacidosis.

8. The nurse is teaching assistive personnel (AP) about hormones that are produced by the adrenal glands. Which hormone has the primary responsibility of maintaining fluid volume and electrolyte composition? a. Sodium b. Magnesium c. Aldosterone d. Renin

ANS: C Aldosterone is a hormone secreted by the adrenal cortex that causes water and sodium absorption to maintain body fluid volume. Renin is secreted by the kidney to trigger angiotensinogen converting angiotensin I to angiotensin II to help control blood pressure. Magnesium and sodium are electrolytes and not hormones.

1. The nurse assesses an older client. What age-related physiologic changes would the nurse expect? a. Heat intolerance b. Rheumatoid arthritis c. Dehydration d. Increased appetite

ANS: C As people age, the many of the endocrine glands decrease hormone production, including a decrease in antidiuretic hormone production. This change, in addition to less body fluid being present as one ages, can cause dehydration. Older adults usually have cold intolerance and a decrease in appetite. Rheumatoid arthritis is not an age-related change; osteoarthritis causes primarily by aging.

8. A nurse cares for a client who has hypothyroidism as a result of Hashimoto thyroiditis. The client asks, "How long will I need to take this thyroid medication?" How would the nurse respond? a. "You will need to take the thyroid medication until the goiter is completely gone." b. "Thyroiditis is cured with antibiotics. Then you won't need thyroid medication." c. "You'll need thyroid pills for life because your thyroid won't start working again." d. "When blood tests indicate normal thyroid function, you can stop the medication."

ANS: C Hashimoto thyroiditis results in a permanent loss of thyroid function. The client will need lifelong thyroid replacement therapy and will not be able to stop taking the medication.

6. A nurse plans care for a client with hypothyroidism. Which priority problem does the nurse address first for this client? a. Heat intolerance b. Body image problems c. Depression and withdrawal d. Obesity and water retention

ANS: C Hypothyroidism causes many problems in psychosocial functioning. Depression is the most common reason for seeking medical attention. Memory and attention span may be impaired. The client's family may have great difficulty accepting and dealing with these changes. The client is often unmotivated to participate in self-care. Lapses in memory and attention require the nurse to ensure that the patient's environment is safe. Heat intolerance is seen in hyperthyroidism. Body image problems and weight issues do not take priority over mental status and safety.

2. A nurse plans care for a client with a growth hormone deficiency. Which action would the nurse include in this client's plan of care? a. Avoid intramuscular medications. b. Place the client in protective isolation. c. Use a lift sheet to reposition the patient. d. Assist the client to dangle before rising.

ANS: C In adults, growth hormone is necessary to maintain bone density and strength. Adults with growth hormone deficiency have thin, fragile bones. Avoiding IM medications, using protective isolation, and assisting the client as he or she moves from sitting to standing will not serve as safety measures when the client is deficient in growth hormone.

3. A nurse assesses a client who is recovering from a subtotal thyroidectomy. On the first postoperative day before discharge, the client states, "I feel numbness and tingling around my mouth." What action does the nurse take? a. Offer mouth care. b. Loosen the dressing. c. Assess for muscle twitching. d. Ask the client orientation questions.

ANS: C Numbness and tingling around the mouth or in the fingers and toes are manifestations of hypocalcemia, which could progress to cause tetany and seizure activity. The nurse would assess for muscle twitching and, if present, notify the surgeon or Rapid Response Team to give calcium gluconate or other IV calcium replacement. Mouth care, loosening the dressing, and orientation questions do not provide important information to prevent complications of low calcium levels.

2. The nurse is assessing a client for risk of developing metabolic syndrome. Which risk factor is associated with this health condition? a. Hypotension b. Hyperthyroidism c. Abdominal obesity d. Hypoglycemia

ANS: C The client at risk for metabolic syndrome typically has hypertension, abdominal obesity, hyperlipidemia, and hyperglycemia.

8. The nurse is caring for a client who is diagnosed with diabetes insipidus (DI). For what common complication will the nurse monitor? a. Hypertension b. Bradycardia c. Dehydration d. Pulmonary embolus

ANS: C The client who has DI has fluid loss through excessive urination. Decreased fluid volume, or dehydration, is manifested by tachycardia, hypotension, and possibly elevated temperature. Pulmonary embolism (PE) could possible as a clot in the lower extremity (caused by dehydration) could fragment and travel to the lungs.

36. The nurse is caring for a newly admitted client who is diagnosed with hyperglycemic-hyperosmolar state (HHS). What is the nurse's priority action at this time? a. Assess the client's blood glucose level. b. Monitor the client's urinary output every hour. c. Establish intravenous access to provide fluids. d. Give regular insulin per agency policy.

ANS: C The first priority in caring for a client with HHS is to increase blood volume to prevent shock or severe hypotension from dehydration. The nurse would monitor vital signs, urinary output, and blood glucose to determine if interventions were effective. Regular insulin is also indicated but not as the first priority action.

12. The nurse is preparing to give tolvaptan for a client who has syndrome of inappropriate antidiuretic hormone (SIADH). For which potentially life-threatening adverse effect would the nurse monitor? a. Increased intracranial pressure b. Myocardial infarction c. Rapid-onset hypernatremia d. Bowel perforation

ANS: C Tolvaptan has a black box warning that rapid increases in serum sodium levels have been associated with central nervous system demyelination that can lead to serious complications and death.

2. A nurse is caring for a client with elevated triiodothyronine and thyroxine, and normal thyroid-stimulating hormone levels. What actions does the nurse take? (Select all that apply.) a. Administer levothyroxine. b. Administer propranolol. c. Monitor the apical pulse. d. Assess for Trousseau sign. e. Initiate telemetry monitoring.

ANS: C,E The client's laboratory findings suggest that the client is experiencing hyperthyroidism. The increased metabolic rate can caGuRseAaDnEinScrLeAasBe.inCtOheMclient's heart rate, and the client should be monitored for the development of dysrhythmias. Placing the client on a telemetry monitor might also be a precaution. Levothyroxine is given for hypothyroidism. Propranolol is a beta blocker often used to lower sympathetic nervous system activity in hyperthyroidism. Trousseau sign is a test for hypocalcemia.

14. A nurse reviews the laboratory test values for a client with a new diagnosis of diabetes mellitus type 2. Which A1C value would the nurse expect? a. 5.0% b. 5.7% c. 6.2% d. 7.4%

ANS: D A client is diagnosed with diabetes if the client's A1C is 6.5% or greater. All listed values are below that level except for 7.4%.

9. The nurse reviews the function of thyroid gland hormones. What is the primary function of calcitonin? a. Sodium and potassium balance b. Magnesium balance c. Norepinephrine balance d. Calcium and phosphorus balance

ANS: D Calcitonin is the primary body hormone that is secreted from the thyroid gland and is responsible for maintaining calcium and phosphorus balance.

4. A nurse assesses a client on the medical-surgical unit. Which statement made by the client alerts the nurse to assess the patient for hypothyroidism? a. "My sister has thyroid problems." b. "I seem to feel the heat more than other people." c. "Food just doesn't taste good without a lot of salt." d. "I am always tired, even with 12 hours of sleep."

ANS: D Clients with hypothyroidism usually feel tired or weak despite getting many hours of sleep. Most thyroid problems are not inherited, although they may occur in families. Heat intolerance is indicative of hyperthyroidism. Loss of taste is not a manifestation of hypothyroidism. The nurse would assess the client further for hypothyroidism.

7. A nurse assesses clients who are at risk for diabetes mellitus. Which client is at greatest risk? a. A 19-year-old Caucasian b. A 22-year-old African American c. A 44-year-old Asian American d. A 58-year-old American Indian

ANS: D Diabetes is a particular problem among African Americans, Hispanics, and American Indians. The incidence of diabetes increases in all races and ethnic groups with age. Being both an American Indian and middle age places this patient at highest risk.

40. A patient is brought to the emergency department by the paramedics. The patient is a type 2 diabetic and is experiencing HHS. The nurse should identify what components of HHS? Select all that apply. A) Leukocytosis B) Glycosuria C) Dehydration D) Hypernatremia E) Hyperglycemia

B, C, D, E Feedback: In HHS, persistent hyperglycemia causes osmotic diuresis, which results in losses of water and electrolytes. To maintain osmotic equilibrium, water shifts from the intracellular fluid space to the extracellular fluid space. With glycosuria and dehydration, hypernatremia and increased osmolarity occur. Leukocytosis does not take place.

37. The nurse is caring for a patient at risk for an addisonian crisis. For what associated signs and symptoms should the nurse monitor the patient? Select all that apply. A) Epistaxis B) Pallor C) Rapid respiratory rate D) Bounding pulse E) Hypotension

B, C, E Feedback: The patient at risk is monitored for signs and symptoms indicative of addisonian crisis, which can include shock; hypotension; rapid, weak pulse; rapid respiratory rate; pallor; and extreme weakness. Epistaxis and a bounding pulse are not symptoms or signs of an addisonian crisis.

25. A patient has been admitted to the critical care unit with a diagnosis of thyroid storm. What interventions should the nurse include in this patients immediate care? Select all that apply. A) Administering diuretics to prevent fluid overload B) Administering beta blockers to reduce heart rate C) Administering insulin to reduce blood glucose levels D) Applying interventions to reduce the patients temperature E) Administering corticosteroids

B, D Feedback: Thyroid storm necessitates interventions to reduce heart rate and temperature. Diuretics, insulin, and steroids are not indicated to address the manifestations of this health problem.

20. The nurse is assessing a patient diagnosed with Graves disease. What physical characteristics of Graves disease would the nurse expect to find? A) Hair loss B) Moon face C) Bulging eyes D) Fatigue

C Feedback: Clinical manifestations of the endocrine disorder Graves disease include exophthalmos (bulging eyes) and fine tremor in the hands. Graves disease is not associated with hair loss, a moon face, or fatigue.

3. A patient newly diagnosed with type 2 diabetes is attending a nutrition class. What general guideline would be important to teach the patients at this class? A) Low fat generally indicates low sugar. B) Protein should constitute 30% to 40% of caloric intake. C) Most calories should be derived from carbohydrates. D) Animal fats should be eliminated from the diet.

C Feedback: Currently, the ADA and the Academy of Nutrition and Dietetics (formerly the American Dietetic Association) recommend that for all levels of caloric intake, 50% to 60% of calories should be derived from carbohydrates, 20% to 30% from fat, and the remaining 10% to 20% from protein.Low fat does not automatically mean low sugar. Dietary animal fat does not need to be eliminated from the diet.

29. A 30 year-old female patient has been diagnosed with Cushing syndrome. What psychosocial nursing diagnosis should the nurse most likely prioritize when planning the patients care? A) Decisional conflict related to treatment options B) Spiritual distress related to changes in cognitive function C) Disturbed body image related to changes in physical appearance D) Powerlessness related to disease progression

C Feedback: Cushing syndrome causes characteristic physical changes that are likely to result in disturbed body image. Decisional conflict and powerless may exist, but disturbed body image is more likely to be present. Cognitive changes take place in patients with Cushing syndrome, but these may or may not cause spiritual distress.

9. The nurse caring for a patient with Cushing syndrome is describing the dexamethasone suppression test scheduled for tomorrow. What does the nurse explain that this test will involve? A) Administration of dexamethasone orally, followed by a plasma cortisol level every hour for 3 hours B) Administration of dexamethasone IV, followed by an x-ray of the adrenal glands C) Administration of dexamethasone orally at 11 PM, and a plasma cortisol level at 8 AM the next morning D) Administration of dexamethasone intravenously, followed by a plasma cortisol level 3 hours after the drug is administered

C Feedback: Dexamethasone (1 mg) is administered orally at 11 PM, and a plasma cortisol level is obtained at 8 AM the next morning. This test can be performed on an outpatient basis and is the most widely used and sensitive screening test for diagnosis of pituitary and adrenal causes of Cushing syndrome.

4. A nurse is providing health education to an adolescent newly diagnosed with type 1 diabetes mellitus and her family. The nurse teaches the patient and family that which of the following nonpharmacologic measures will decrease the bodys need for insulin? A) Adequate sleep B) Low stimulation C) Exercise D) Low-fat diet

C Feedback: Exercise lowers blood glucose, increases levels of HDLs, and decreases total cholesterol and triglyceride levels. Low fat intake and low levels of stimulation do not reduce a patients need for insulin. Adequate sleep is beneficial in reducing stress, but does not have an effect that is pronounced as that of exercise.

35. A patient with type 1 diabetes mellitus is seeing the nurse to review foot care. What would be a priority instruction for the nurse to give the patient? A) Examine feet weekly for redness, blisters, and abrasions. B) Avoid the use of moisturizing lotions. C) Avoid hot-water bottles and heating pads. D) Dry feet vigorously after each bath.

C Feedback: High-risk behaviors, such as walking barefoot, using heating pads on the feet, wearing open-toed shoes, soaking the feet, and shaving calluses, should be avoided. Socks should be worn for warmth. Feet should be examined each day for cuts, blisters, swelling, redness, tenderness, and abrasions. Lotion should be applied to dry feet but never between the toes. After a bath, the patient should gently, not vigorously, pat feet dry to avoid injury.

20. A 28-year-old pregnant woman is spilling sugar in her urine. The physician orders a glucose tolerance test, which reveals gestational diabetes. The patient is shocked by the diagnosis, stating that she is conscientious about her health, and asks the nurse what causes gestational diabetes. The nurse should explain that gestational diabetes is a result of what etiologic factor? A) Increased caloric intake during the first trimester B) Changes in osmolality and fluid balance C) The effects of hormonal changes during pregnancy D) Overconsumption of carbohydrates during the first two trimesters

C Feedback: Hyperglycemia and eventual gestational diabetes develops during pregnancy because of the secretion of placental hormones, which causes insulin resistance. The disease is not the result of food intake or changes in osmolality.

32. A student with diabetes tells the school nurse that he is feeling nervous and hungry. The nurse assesses the child and finds he has tachycardia and is diaphoretic with a blood glucose level of 50 mg/dL (2.8 mmol/L). What should the school nurse administer? A) A combination of protein and carbohydrates, such as a small cup of yogurt B) Two teaspoons of sugar dissolved in a cup of apple juice C) Half of a cup of juice, followed by cheese and crackers D) Half a sandwich with a protein-based filling

C Feedback: Initial treatment for hypoglycemia is 15 g concentrated carbohydrate, such as two or three glucose tablets, 1 tube glucose gel, or 0.5 cup juice. After initial treatment, the nurse should follow with a snack including starch and protein, such as cheese and crackers, milk and crackers, or half of a sandwich. It is unnecessary to add sugar to juice, even it if is labeled as unsweetened juice, because the fruit sugar in juice contains enough simple carbohydrate to raise the blood glucose level and additional sugar may result in a sharp rise in blood sugar that will last for several hours.

16. A patient has returned to the floor after having a thyroidectomy for thyroid cancer. The nurse knows that sometimes during thyroid surgery the parathyroid glands can be injured or removed. What laboratory finding may be an early indication of parathyroid gland injury or removal? A) Hyponatremia B) Hypophosphatemia C) Hypocalcemia D) Hypokalemia

C Feedback: Injury or removal of the parathyroid glands may produce a disturbance in calcium metabolism and result in a decline of calcium levels (hypocalcemia). As the blood calcium levels fall, hyperirritability of the nerves occurs, with spasms of the hands and feet and muscle twitching. This group of symptoms is known as tetany and must be reported to the physician immediately, because laryngospasm may occur and obstruct the airway. Hypophosphatemia, hyponatremia, and hypokalemia are not expected responses to parathyroid injury or removal. In fact, parathyroid removal or injury that results in hypocalcemia may lead to hyperphosphatemia.

13. A diabetes educator is teaching a patient about type 2 diabetes. The educator recognizes that the patient understands the primary treatment for type 2 diabetes when the patient states what? A) I read that a pancreas transplant will provide a cure for my diabetes. B) I will take my oral antidiabetic agents when my morning blood sugar is high. C) I will make sure to follow the weight loss plan designed by the dietitian. D) I will make sure I call the diabetes educator when I have questions about my insulin.

C Feedback: Insulin resistance is associated with obesity; thus the primary treatment of type 2 diabetes is weight loss. Oral antidiabetic agents may be added if diet and exercise are not successful in controlling blood glucose levels. If maximum doses of a single category of oral agents fail to reduce glucose levels to satisfactory levels, additional oral agents may be used. Some patients may require insulin on an ongoing basis or on a temporary basis during times of acute psychological stress, but it is not the central component of type 2 treatment. Pancreas transplantation is associated with type 1 diabetes.

1. A patient with type 1 diabetes has told the nurse that his most recent urine test for ketones was positive. What is the nurses most plausible conclusion based on this assessment finding? A) The patient should withhold his next scheduled dose of insulin. B) The patient should promptly eat some protein and carbohydrates. C) The patients insulin levels are inadequate. D) The patient would benefit from a dose of metformin (Glucophage).

C Feedback: Ketones in the urine signal that there is a deficiency of insulin and that control of type 1 diabetes is deteriorating. Withholding insulin or eating food would exacerbate the patients ketonuria. Metformin will not cause short-term resolution of hyperglycemia.

4. The nurse is caring for a patient with a diagnosis of Addisons disease. What sign or symptom is most closely associated with this health problem? A) Truncal obesity B) Hypertension C) Muscle weakness D) Moon face

C Feedback: Patients with Addisons disease demonstrate muscular weakness, anorexia, gastrointestinal symptoms, fatigue, emaciation, dark pigmentation of the skin, and hypotension. Patients with Cushing syndrome demonstrate truncal obesity, moon face, acne, abdominal striae, and hypertension.

8. A nurse caring for a patient with diabetes insipidus is reviewing laboratory results. What is an expected urinalysis finding? A) Glucose in the urine B) Albumin in the urine C) Highly dilute urine D) Leukocytes in the urine

C Feedback: Patients with diabetes insipidus produce an enormous daily output of very dilute, water-like urine with a specific gravity of 1.001 to 1.005. The urine contains no abnormal substances such as glucose or albumin. Leukocytes in the urine are not related to the condition of diabetes insipidus, but would indicate a urinary tract infection, if present in the urine.

24. A physician has explained to a patient that he has developed diabetic neuropathy in his right foot. Later that day, the patient asks the nurse what causes diabetic neuropathy. What would be the nurses best response? A) Research has shown that diabetic neuropathy is caused by fluctuations in blood sugar that have gone on for years. B) The cause is not known for sure but it is thought to have something to do with ketoacidosis. C) The cause is not known for sure but it is thought to involve elevated blood glucose levels over a period of years. D) Research has shown that diabetic neuropathy is caused by a combination of elevated glucose levels and elevated ketone levels.

C Feedback: The etiology of neuropathy may involve elevated blood glucose levels over a period of years. High blood sugars (rather than fluctuations or variations in blood sugars) are thought to be responsible. Ketones and ketoacidosis are not direct causes of neuropathies.

6. The nurse is teaching a patient that the body needs iodine for the thyroid to function. What food would be the best source of iodine for the body? A) Eggs B) Shellfish C) Table salt D) Red meat

C Feedback: The major use of iodine in the body is by the thyroid. Iodized table salt is the best source of iodine.

7. A patient is prescribed corticosteroid therapy. What would be priority information for the nurse to give the patient who is prescribed long-term corticosteroid therapy? A) The patients diet should be low protein with ample fat. B) The patient may experience short-term changes in cognition. C) The patient is at an increased risk for developing infection. D) The patient is at a decreased risk for development of thrombophlebitis and thromboembolism.

C Feedback: The patient is at increased risk of infection and masking of signs of infection. The cardiovascular effects of corticosteroid therapy may result in development of thrombophlebitis or thromboembolism. Diet should be high in protein with limited fat. Changes in appearance usually disappear when therapy is no longer necessary. Cognitive changes are not common adverse effects.

19. Which of the following patients with type 1 diabetes is most likely to experience adequate glucose control? A) A patient who skips breakfast when his glucose reading is greater than 220 mg/dL B) A patient who never deviates from her prescribed dose of insulin C) A patient who adheres closely to a meal plan and meal schedule D) A patient who eliminates carbohydrates from his daily intake

C Feedback: The therapeutic goal for diabetes management is to achieve normal blood glucose levels without hypoglycemia. Therefore, diabetes management involves constant assessment and modification of the treatment plan by health professionals and daily adjustments in therapy (possibly including insulin) by patients. For patients who require insulin to help control blood glucose levels, maintaining consistency in the amount of calories and carbohydrates ingested at meals is essential. In addition, consistency in the approximate time intervals between meals, and the snacks, help maintain overall glucose control. Skipping meals is never advisable for person with type 1 diabetes.

2. A patient has been admitted to the post-surgical unit following a thyroidectomy. To promote comfort and safety, how should the nurse best position the patient? A) Side-lying (lateral) with one pillow under the head B) Head of the bed elevated 30 degrees and no pillows placed under the head C) Semi-Fowlers with the head supported on two pillows D) Supine, with a small roll supporting the neck

C Feedback: When moving and turning the patient, the nurse carefully supports the patients head and avoids tension on the sutures. The most comfortable position is the semi-Fowlers position, with the head elevated and supported by pillows.

38. A patient has been assessed for aldosteronism and has recently begun treatment. What are priority areas for assessment that the nurse should frequently address? Select all that apply. A) Pupillary response B) Creatinine and BUN levels C) Potassium level D) Peripheral pulses E) BP

C, E Feedback: Patients with aldosteronism exhibit a profound decline in the serum levels of potassium, and hypertension is the most prominent and almost universal sign of aldosteronism. Pupillary response, peripheral pulses, and renal function are not directly affected.

38. A nurse is assessing a patient who has diabetes for the presence of peripheral neuropathy. The nurse should question the patient about what sign or symptom that would suggest the possible development of peripheral neuropathy? A) Persistently cold feet B) Pain that does not respond to analgesia C) Acute pain, unrelieved by rest D) The presence of a tingling sensation

D Feedback: Although approximately half of patients with diabetic neuropathy do not have symptoms, initial symptoms may include paresthesias (prickling, tingling, or heightened sensation) and burning sensations (especially at night). Cold and intense pain are atypical early signs of this complication.

31. A patient has received a diagnosis of type 2 diabetes. The diabetes nurse has made contact with the patient and will implement a program of health education. What is the nurses priority action? A) Ensure that the patient understands the basic pathophysiology of diabetes. B) Identify the patients body mass index. C) Teach the patient survival skills for diabetes. D) Assess the patients readiness to learn.

D Feedback: Before initiating diabetes education, the nurse assesses the patients (and familys) readiness to learn. This must precede other physiologic assessments (such as BMI) and providing health education.

27. A patient has been taking prednisone for several weeks after experiencing a hypersensitivity reaction. To prevent adrenal insufficiency, the nurse should ensure that the patient knows to do which of the following? A) Take the drug concurrent with levothyroxine (Synthroid). B) Take each dose of prednisone with a dose of calcium chloride. C) Gradually replace the prednisone with an OTC alternative. D) Slowly taper down the dose of prednisone, as ordered.

D Feedback: Corticosteroid dosages are reduced gradually (tapered) to allow normal adrenal function to return and to prevent steroid-induced adrenal insufficiency. There are no OTC substitutes for prednisone and neither calcium chloride nor levothyroxine addresses the risk of adrenal insufficiency.

19. A patient with hypofunction of the adrenal cortex has been admitted to the medical unit. What would the nurse most likely find when assessing this patient? A) Increased body temperature B) Jaundice C) Copious urine output D) Decreased BP

D Feedback: Decreased BP may occur with hypofunction of the adrenal cortex. Decreased function of the adrenal cortex does not affect the patients body temperature, urine output, or skin tone.

21. A medical nurse is aware of the need to screen specific patients for their risk of hyperglycemic hyperosmolar syndrome (HHS). In what patient population does hyperosmolar nonketotic syndrome most often occur? A) Patients who are obese and who have no known history of diabetes B) Patients with type 1 diabetes and poor dietary control C) Adolescents with type 2 diabetes and sporadic use of antihyperglycemics D) Middle-aged or older people with either type 2 diabetes or no known history of diabetes

D Feedback: HHS occurs most often in older people (50 to 70 years of age) who have no known history of diabetes or who have type 2 diabetes.

24. A patient with a recent diagnosis of hypothyroidism is being treated for an unrelated injury. When administering medications to the patient, the nurse should know that the patients diminished thyroid function may have what effect? A) Anaphylaxis B) Nausea and vomiting C) Increased risk of drug interactions D) Prolonged duration of effect

D Feedback: In all patients with hypothyroidism, the effects of analgesic agents, sedatives, and anesthetic agents are prolonged. There is no direct increase in the risk of anaphylaxis, nausea, or drug interactions, although these may potentially result from the prolonged half-life of drugs.

12. The home care nurse is conducting patient teaching with a patient on corticosteroid therapy. To achieve consistency with the bodys natural secretion of cortisol, when would the home care nurse instruct the patient to take his or her corticosteroids? A) In the evening between 4 PM and 6 PM B) Prior to going to sleep at night C) At noon every day D) In the morning between 7 AM and 8 AM

D Feedback: In keeping with the natural secretion of cortisol, the best time of day for the total corticosteroid dose is in the morning from 7 to 8 AM. Large-dose therapy at 8 AM, when the adrenal gland is most active, produces maximal suppression of the gland. Also, a large 8 AM dose is more physiologic because it allows the body to escape effects of the steroids from 4 PM to 6 AM, when serum levels are normally low, thus minimizing cushingoid effects.

29. A nurse is teaching basic survival skills to a patient newly diagnosed with type 1 diabetes. What topic should the nurse address? A) Signs and symptoms of diabetic nephropathy B) Management of diabetic ketoacidosis C) Effects of surgery and pregnancy on blood sugar levels D) Recognition of hypoglycemia and hyperglycemia

D Feedback: It is imperative that newly diagnosed patients know the signs and symptoms and management of hypo- and hyperglycemia. The other listed topics are valid points for education, but are not components of the patients immediate survival skills following a new diagnosis.

25. A patient with type 2 diabetes has been managing his blood glucose levels using diet and metformin (Glucophage). Following an ordered increase in the patients daily dose of metformin, the nurse should prioritize which of the following assessments? A) Monitoring the patients neutrophil levels B) Assessing the patient for signs of impaired liver function C) Monitoring the patients level of consciousness and behavior D) Reviewing the patients creatinine and BUN levels

D Feedback: Metformin has the potential to be nephrotoxic; consequently, the nurse should monitor the patients renal function. This drug does not typically affect patients neutrophils, liver function, or cognition.

9. A school nurse is teaching a group of high school students about risk factors for diabetes. Which of the following actions has the greatest potential to reduce an individuals risk for developing diabetes? A) Have blood glucose levels checked annually. B) Stop using tobacco in any form. C) Undergo eye examinations regularly. D) Lose weight, if obese.

D Feedback: Obesity is a major modifiable risk factor for diabetes. Smoking is not a direct risk factor for the disease. Eye examinations are necessary for persons who have been diagnosed with diabetes, but they do not screen for the disease or prevent it. Similarly, blood glucose checks do not prevent the diabetes.

34. A patient on corticosteroid therapy needs to be taught that a course of corticosteroids of 2 weeks duration can suppress the adrenal cortex for how long? A) Up to 4 weeks B) Up to 3 months C) Up to 9 months D) Up to 1 year

D Feedback: Suppression of the adrenal cortex may persist up to 1 year after a course of corticosteroids of only 2 weeks duration.

15. An older adult patient with type 2 diabetes is brought to the emergency department by his daughter. The patient is found to have a blood glucose level of 623 mg/dL. The patients daughter reports that the patient recently had a gastrointestinal virus and has been confused for the last 3 hours. The diagnosis of hyperglycemic hyperosmolar syndrome (HHS) is made. What nursing action would be a priority? A) Administration of antihypertensive medications B) Administering sodium bicarbonate intravenously C) Reversing acidosis by administering insulin D) Fluid and electrolyte replacement

D Feedback: The overall approach to HHS includes fluid replacement, correction of electrolyte imbalances, and insulin administration. Antihypertensive medications are not indicated, as hypotension generally accompanies HHS due to dehydration. Sodium bicarbonate is not administered to patients with HHS, as their plasma bicarbonate level is usually normal. Insulin administration plays a less important role in the treatment of HHS because it is not needed for reversal of acidosis, as in diabetic ketoacidosis (DKA).

11. A newly admitted patient with type 1 diabetes asks the nurse what caused her diabetes. When the nurse is explaining to the patient the etiology of type 1 diabetes, what process should the nurse describe? A) The tissues in your body are resistant to the action of insulin, making the glucose levels in your blood increase. B) Damage to your pancreas causes an increase in the amount of glucose that it releases, and there is not enough insulin to control it. C) The amount of glucose that your body makes overwhelms your pancreas and decreases your production of insulin. D) Destruction of special cells in the pancreas causes a decrease in insulin production. Glucose levels rise because insulin normally breaks it down.

D Feedback: Type 1 diabetes is characterized by the destruction of pancreatic beta cells, resulting in decreased insulin production, unchecked glucose production by the liver, and fasting hyperglycemia. Also, glucose derived from food cannot be stored in the liver and remains circulating in the blood, which leads to postprandial hyperglycemia. Type 2 diabetes involves insulin resistance and impaired insulin secretion. The body does not make glucose.


Related study sets

Chapter 1. Ethical Dilemmas and Decisions In

View Set

Nurs 125- Chapter 32: Cholinergic Agonists PrepU

View Set

Astronomy Ch04.8: Ranking Task: Gravity and Newton's Laws

View Set

Chapter 29 Study Guide ellenwood

View Set

Chapter 29: Introduction to the Autonomic Nervous System

View Set

ZHAW - Strategisches Management - MC

View Set